Récurrence

Bonjour.
Je considère la suite définie par $$\begin{cases}
u_0&=\ 2\\
u_{n+1}&=\ 1+\dfrac{1}{u_n}
\end{cases}
$$ Je pose $\ \mathcal{P}(n):\quad\dfrac{3}{2}\leq u_n\leq 2.$
Je suppose $\mathcal{P}(n)$ pour un certain entier naturel $n$.
Au rang $n+1$ j'obtiens : $\quad\dfrac{3}{2}\leq u_{n+1}\leq \dfrac{5}{3}$.
La borne supérieure a diminué.
Au rang $n+2$ j'obtiens : $\quad\dfrac{8}{5}\leq u_{n+2}\leq \dfrac{5}{3}$.
La borne inférieure a augmenté.

De rang en rang soit on diminue la borne sup soit on augmente la borne inf.
La suite a l'air de converger. Mais je ne vois pas comment calculer sa limite exacte. J'ai essayé de trouver la forme explicite en calculant les premiers termes mais je ne la trouve pas.
Pouvez-vous m'éclairer ?
Merci :-)

Réponses

  • Supposons que ta suite converge vers une limite $\ell \in [3/2, 2]$. Qu'arrive-t-il quand on passe à la limite dans la relation de récurrence $u_{n+1}=1+\dfrac1{u_n}$ ?
  • « La borne supérieure a augmenté. » Ce n'est pas trop bien dit... Mais ce qui compte, c'est de savoir si $\mathcal{P}(n+1)$ est vraie, c'est-à-dire si on a bien $\frac32\le u_{n+1}\le2$. C'est vrai ou pas ?

    Il serait très parlant de faire un dessin : d'abord le graphe des fonctions $x\mapsto 1+1/x$ et $x\mapsto x$, puis par des constructions simples, les points $(u_0,0)$, $(u_0,u_1)$, $(u_1,u_1)$, $(u_1,0)$, puis $(u_1,u_2)$, $(u_2,u_2)$, $(u_2,0)$, $(u_2,u_3)$, etc.
  • Supposons que $(u_n)$ converge vers $l\in[3/2,2]$.
    On a $\lim u_{n+1}=\lim (1+1/u_n)$ soit $l=1+1/l$ ou encore $l^2-l-1=0$ ce qui nous donne (puisque que l est positif) :

    $$l=\dfrac{1+\sqrt{5}}{2}$$
  • Oui $\mathcal{P}(n+1)$ est vraie puisque 5/3<6/3=2.
  • En dessinant les fonctions $x \rightarrow 1+1/x$ et $x \rightarrow x$ on voit que la suite converge vers le point fixe solution de l'équation $x=1/x+1$.
    Comment montre-t-on que 2 est dans le bassin d'attraction du point fixe $\left(1+\sqrt{5}\right)/2$ ?
  • En essayant de trouver une relation entre $|u_{n+1}-\ell|$ et $|u_n-\ell|$ ?
  • Il serait bien de savoir au juste quelle est la question posée, et à quel niveau il est demandé de la résoudre.
    En particulier, on peut donner l'expression de $u_n$ en fonction de $u_0$ et de $n$, comme pour toute suite homographique.
  • Troisième approche standard : le graphe doit suggérer que la sous-suite formée des termes d'indices pairs (resp. impairs) est monotone.
  • En suivant l'idée de Math Cross :

    On peut montrer par récurrence que la suite des termes d'indices pairs est décroissante et minorée par $\left(1+\sqrt{5}\right)/2$ et que la suite des termes d'indices impairs est croissante et majorée par $\left(1+\sqrt{5}\right)/2$. Donc ces deux suites convergent.
    De plus :

    $$u_{2n+2}=1+\dfrac{u_{2n}}{u_{2n}+1}$$

    Donc en notant $l$ la limite de la suite des termes d'indices pairs on a que

    $$l=1+\dfrac{l}{l+1}\text{ soit } l^2-l-1=0 \text{ soit } l=\left(1+\sqrt{5}\right)/2$$

    De même

    $$u_{2n+3}=1+\dfrac{u_{2n+1}}{u_{2n+1}+1}$$

    Donc en notant $l'$ la limite de la suite des termes d'indices impairs on a que

    $$l'=1+\dfrac{l'}{l'+1}\text{ soit } l'^2-l'-1=0 \text{ soit } l'=\left(1+\sqrt{5}\right)/2$$

    Donc $l=l'$ et la suite $(u_n)$ converge.

    Par contre GaBuZoMeu, je n'ai pas trouvé de lien entre $|u_{n+1}-l|$ et $|u_{n}-l|$.
  • Peut-être en écrivant, avec $f(x)=1+1/x$, $u_{n+1}-\ell=f(u_n)-f(\ell)$ ?
  • OK. On note $l$ l'unique solution de $f(x)=x$. Alors :

    \begin{align}
    |u_{n+1}-l|=|f(u_n)-f(l)|&\leq \left(\max_{[3/2,2]}|f'(x)|\right)|u_n-l|\\
    &\leq \left(\max_{[3/2,2]}\dfrac{1}{x^2}\right)|u_n-l|\\
    &\leq \dfrac{4}{9}|u_n-l|\\
    \end{align}

    Donc
    $$|u_{n}-l|\leq \left(\dfrac{4}{9}\right)^n|u_0-l|$$

    Donc $|u_{n}-l|\rightarrow 0$ et $u_n\rightarrow l$.
  • Il me reste à comprendre comment l'on trouve la formule explicite. :-D
  • Expression du terme général.
    Notre suite s'écrit : $u_{n+1}=f(u_{n})$, avec : $f(x)=1+\frac{1}{x}$.
    On cherche les points fixes de $f$, c'est-à-dire les solutions de l'équation : $f(x)=x$.
    Ces points fixes sont : $\alpha =\frac{1+\sqrt{5}}{2}$ et $\beta =\frac{1-\sqrt{5}}{2}$. Ce sont d'ailleurs les candidats-limites.
    Quels que soient les réels $u$ et $v$ non nuls on a : $f(u)-f(v)=\frac{v-u}{uv}$.
    En particulier : $f(x)-\alpha =f(x)-f(\alpha )=\frac{\alpha -x}{\alpha x}$, $f(x)-\beta =f(x)-f(\beta )=\frac{\beta -x}{\beta x}$.
    Il en résulte : $\frac{f(x)-\alpha }{f(x)-\beta }=\frac{\beta }{\alpha }\cdot \frac{x-\alpha }{x-\beta }$.
    Si l'on pose : $v_{n}=\frac{u_{n}-\alpha }{u_{n}-\beta }$, il vient : $v_{n+1}=\frac{\beta }{\alpha }v_{n}$, suite géométrique. Etc.
    Cette méthode convient pour toute suite réelle ou complexe, à récurrence homographique, ayant deux points fixes, réels ou complexes.
    Bonne soirée.
    Fr. Ch.
    13/09/2017
  • Bien !
    Dans ce cas très précis où la fonction est très simple, on aurait aussi pu écrire
    \[u_{n+1}-\ell=\frac{1}{u_n}-\frac{1}\ell=-\frac{u_n-\ell}{u_n\ell}\] pour aboutir à la même conclusion avec $u_n\ge3/2$ et $\ell\ge3/2$.

    Pour une formule explicite, voici une astuce systématique pour une suite définie par une récurrence de la forme $u_{n+1}=\dfrac{au_n+b}{cu_n+d}$ (dite « homographique »).
    Flash-back: tu as résolu l'équation $f(x)=x$ et trouvé une seule solution $\ell$ mais c'est que tu en as écarté une autre, disons $\ell'$, parce qu'elle n'est pas dans le bon intervalle. Il t'a été très utile de calculer $u_{n+1}-\ell$. Il est aussi très profitable de calculer $\dfrac{u_{n+1}-\ell}{u_{n+1}-\ell'}$.

    Edit : Grillé par Chaurien. Et une précision : « astuce systématique » ça veut dire que présentée ainsi, elle semble artificielle / parachutée ; mais que comme elle marche pour toute suite homographique, c'est une méthode. Avec un peu d'algèbre linéaire, on peut la rendre naturelle.
  • Merci beaucoup, j'ai tout compris (tu)
Connectez-vous ou Inscrivez-vous pour répondre.